Simple ODE problem, Bernoulli's Equation

Click For Summary
The discussion revolves around solving a simple ordinary differential equation (ODE) using Bernoulli's Equation, specifically the initial value problem t*y' - 2*t^2*sqrt(y) = 4*y with y(1) = 4. The user attempts to convert the Bernoulli-type equation into a first-order linear form and applies the integration method for solutions. Despite repeated efforts, the user finds discrepancies when substituting their solution back into the original equation. Ultimately, the issue was identified as a mistake in multiplying both sides of the equation correctly. The user plans to mark the thread as "solved" after resolving the confusion.
Jonnyb42
Messages
185
Reaction score
0
[SOLVED] simple ODE problem, Bernoulli's Equation

Homework Statement



Initial value problem:

Relation: t*y' - 2*[t^2]*sqrt(y) = 4*y
Initial value: y(1) = 4

Homework Equations



general form of Bernoulli's equation:
y' + a(t)y = b(t)*[y^n]

First order, linear ODE form:
y' + a(t)y = b(t)


The Attempt at a Solution



My written solution. I first get Bernoulli-type equation into first order/linear form. After that I solve it with the equation y = [1/mu]*Integral[ b(t) * mu dt] (+ constant)
where mu = e^[ Integral[ a(t) dt]

27zf728.jpg


I have tried this multiple times and I get the same answer. When I plug in the solution y = f(t) it does not match the differential equation, (takes some time to show.)

Any help would be great, I obviously am doing something wrong.
 
Last edited:
Physics news on Phys.org
Well it turns out I forgot how to multiply both sides of an equation.
3rd to last step I multiply half of the right side by t^2, I'm not sure how to make this thread "solved."
 
Jonnyb42 said:
I'm not sure how to make this thread "solved."

I believe you can edit the header of your own posts, so you can put a " [SOLVED] " at the end of your title.
 
k thanks ill do that
 
Question: A clock's minute hand has length 4 and its hour hand has length 3. What is the distance between the tips at the moment when it is increasing most rapidly?(Putnam Exam Question) Answer: Making assumption that both the hands moves at constant angular velocities, the answer is ## \sqrt{7} .## But don't you think this assumption is somewhat doubtful and wrong?

Similar threads

  • · Replies 3 ·
Replies
3
Views
2K
Replies
3
Views
2K
  • · Replies 3 ·
Replies
3
Views
1K
Replies
9
Views
2K
  • · Replies 11 ·
Replies
11
Views
2K
Replies
4
Views
1K
  • · Replies 5 ·
Replies
5
Views
2K
  • · Replies 14 ·
Replies
14
Views
3K
Replies
7
Views
2K
  • · Replies 33 ·
2
Replies
33
Views
4K